Arithmogon Rechteckig

Aufrufe: 1409     Aktiv: 18.10.2018 um 12:32

0
Und schon die nächste Frage. Wir haben ein  rechteckiges Arithmogon bekommen, wo die Eckzahlen unbekannt sind. es ergibt sich folgendes Gleichungssystem (meiner Meinung nach): w+x= 18 x+z= 47 z+y= 52 y+w=23 jetzt sollen wir alle Möglichkeiten für die Eckzahlen w,x,y und z bestimmen. Meine Frage ist jetzt wie das gehen soll?   Im zweiten Teil sollen wir dann den allgemeinen Fall betrachten, Eckzahlen sind unbekannt, die anderen bekannt, können sogar gleich sein. Dazu soll ein LGS aufgestellt werden und ermittelt werden wann es genau eine Lösung gibt, unendlich viele Lösungen und keine Lösung jeweils mit der expliziten Lösungsmenge. Ich bin leider total überfordert und ratlos..
Diese Frage melden
gefragt

Student, Punkte: 163

 
Kommentar schreiben
4 Antworten
0
Hallo, bei dir ist die Reihenfolge von w,x,y und z immer von links nach rechts oder? Also oben links w oben rechts x unten links y unten rechts z. Hatte zuerst die Ecken im Uhrzeigersinn beschriftet deshalb war ich kurz verwirrt :p Wenn du das Gleichungssystem löst siehst du, das es unendlich viele Lösungen gibt. Das sieht folgendermaßen aus: \( x+z = 47 \Rightarrow x = 47 - z \) eingesetzt in die erste Gleichung ergibt das \(  w+x = w+47-z = 18 \Rightarrow w-z = -29 \) \( z+y=52 \Rightarrow y=52-z \) eingesetzt in die letzte Gleichung ergibt das \( y+w= 52-z +w = 23 \Rightarrow w-z = -29 \) Subtrahiert man beide Gleichungen erhält man 0=0 . Wir müssen also eine Variable frei wählen. Sagen wir w = t . Dann folgt : \( w = t \) \( x = 18 -t \) \( z = 47 - x = 47 -18 + t = 29 + t \) \( y = 23 - t \) Unser Lösungsvektor ergibt sich dann zu \( \begin{bmatrix} w \\ x \\ y \\ z \end{bmatrix} = \begin{bmatrix} t \\ 18-t \\ 23-t \\ 29+t \end{bmatrix} = \begin{bmatrix} 0 \\ 18 \\ 23 \\ 29 \end{bmatrix} + t \begin{bmatrix} 1 \\ -1 \\ -1 \\ 1 \end{bmatrix} \) Die allgemeine Berechnung verläuft analog. In dem Schritt in dem ich gezeigt habe das es unendlich viele Lösungen gibt, kannst du allgemein zeigen wann es unendlich viele Lösungen und wann es keine Lösung gibt. Nimm dafür die Gleichungen \( w+x = a \) \( x+z = b \) \( z+y= c \) \( y+w = d \) a,b,c und d können nach belieben für die anderen Zahlen gesetzt werden. Habe jetzt einfach die Gleichungen von oben genommen und dort in der Reihenfolge a,b,c und d gesetzt. Versuch mal die allgemeine Lösung zu berechnen. Bei Rückfragen oder Unklarheiten melde dich nochmal Grüße Christian
Diese Antwort melden
geantwortet

Sonstiger Berufsstatus, Punkte: 29.81K

 

Kommentar schreiben

0
Christian, du bist ein Genie! Da wäre ich nie im Leben drauf gekommen... für das allgemeine bin ich jetzt soweit gekommen: (w        (    a-b+z x   =      b-z y            c-z z)            -d+c+w)   also a+t =w b-t =x c-t = y d+t=z richtig? Aber irgendwie führt das zu nichts.. bzw. ich glaube ich bin auf dem Holzweg.
Diese Antwort melden
geantwortet

Student, Punkte: 163

 

Kommentar schreiben

0

Soweit würde ich nicht gehen hören tue ich es trotzdem gerne :p

Wenn ich das richtig sehe hast du im oberen Teil:

\( w-z = a-b \)

\( w-z = d-c \)

Das stimmt soweit schon mal.

Daraus bestimmen wir für welche a,b,c und d das ganze lösbar ist oder eben nicht. Wenn wir von der ersten Gleichung die zweite subtrahieren kommen wir auf

\( 0= a-b-(d-c)= a-b+c-d \)

Das ist unsere Einschränkung. Die Möglichkeit nur eine einzige Lösung gibt es nicht. Es gibt entweder unendlich viele Lösungen oder eben keine.

Keine Lösung haben wir zum Beispiel bei \( a=1,b=2,c=3,d=4\) da \( 1-2+3-4=-2 \neq 0 \)

\(a-b+c-d =0 \) muss erfüllt sein.

Nehmen wir an a,b,c und d erfüllen diese Gleichung.

Dann müssen wir wieder eine der Variablen w,x,y und z frei wählen. Das entsteht nicht durch umstellen einer Gleichung. Nehmen wir wieder

\( w=t \)

Daraus folgt durch einsetzen von \( w=t \) in die erste und vierte Gleichung sofort

\( x +t = a \Rightarrow x= a-t \)

\( y+ t = d \Rightarrow y = d-t \)

Bis hierhin hast du das richtig gemacht. Nur der Wert für z stimmt nicht ganz, denn

\( z + y = z +d - t = c \Rightarrow z = (c-d) + t \)

Jetzt musst du nur noch den Lösungsvektor aufstellen.

\( \begin{bmatrix} w \\ x \\ y \\ z \end{bmatrix} = \begin{bmatrix} t \\ a-t \\ d-t \\ (c-d)+t \end{bmatrix} = \begin{bmatrix} 0 \\ a\\ d\\ (c-d) \end{bmatrix} + t \begin{bmatrix} 1 \\ -1 \\ -1 \\ 1 \end{bmatrix} \)

Du warst also fast fertig ;)

Alles verständlich?

Zur Probe kannst du die Zahlen aus dem ersten Teil für a,b,c und d einsetzen. Es muss die Lösung aus dem ersten Teil herauskommen. :)

Grüße Christian

 

Diese Antwort melden
geantwortet

Sonstiger Berufsstatus, Punkte: 29.81K

 

Kommentar schreiben

0
Ok jetzt hab ich’s! Tausend Dank!!!!!
Diese Antwort melden
geantwortet

Student, Punkte: 163

 

Sehr gerne.   ─   christian_strack 18.10.2018 um 18:24

Kommentar schreiben